สนาม E และ B อยู่ในระยะของรังสีแม่เหล็กไฟฟ้าหรือไม่?


10

ฉันเพิ่งเขียนคำตอบนี้ซึ่งฉันพูดว่า:

คลื่นวิทยุเป็นแม่เหล็กไฟฟ้ารังสี รังสีแม่เหล็กไฟฟ้าประกอบด้วยสององค์ประกอบหนึ่งไฟฟ้าและหนึ่งแม่เหล็ก ส่วนประกอบเหล่านี้สร้างซึ่งกันและกันดังที่ได้กล่าวข้างต้น สนามแม่เหล็กสีแดงสร้างสนามไฟฟ้าสีน้ำเงินซึ่งสร้างสนามแม่เหล็กถัดไปและอื่น ๆ

ป้อนคำอธิบายรูปภาพที่นี่

ฉันได้รับแผนภาพนี้จากวิกิพีเดีย แต่หนังสือฟิสิกส์ของฉันและจิมฮอว์กินส์ WA2WHVให้แผนภาพเดียวกัน

ในความคิดเห็นการสนทนาตาม:

แลงลาทรอป : แผนภาพแรกของคุณผิด จริง ๆ แล้วฟิลด์ B และ E นั้นมีระยะห่างกัน 90 องศาไม่ใช่เฟสที่แสดง พลังงานพุ่งอย่างต่อเนื่องไปมาระหว่างเขต E และ B

Keelan : แน่ใจนะ? Wikipedia และหนังสือฟิสิกส์ของฉันแสดงต่างออกไป เขตข้อมูลทั้งสองควรมีอัตราส่วนคงที่ฉันเชื่อว่าไม่สามารถเกิดขึ้นได้เมื่ออยู่นอกเฟส ฟิลด์หนึ่งเป็นแนวนอนและอีกแนวตั้งมีมุม 90 องศา - แผนภาพคือความพยายามในการแสดงสามมิติ

Olin Lathrop : อืม ฉันเข้าใจพวกเขาเสมอในการสร้างพื้นที่สี่เหลี่ยมจัตุรัส แต่ฉันไม่มีเวลาที่จะค้นหามันในตอนนี้ นี่อาจเป็นกรณีของไดอะแกรมที่ไม่ดีอย่างหนึ่งที่ถูกคัดลอกโดยคนอื่น ๆ จำนวนมาก พลังงานอยู่ที่ไหนเมื่อทั้งสองฟิลด์กด 0 ในแผนภาพของคุณ? ในการสร้างพื้นที่สี่เหลี่ยมจัตุรัสผลรวมของกำลังสองของแต่ละแอมพลิจูดของสนามนั้นเป็นค่าคงที่ซึ่งให้คำอธิบายที่ดีว่าพลังงานสามารถคงอยู่ได้อย่างไร มันลื่นไถลไปมาระหว่างสองฟิลด์ แต่ผลรวมของมันจะเท่ากันเสมอ

ฉันทำตามตรรกะของแลงและไม่สามารถบอกตัวเองได้ว่าทำไมทุ่งถึงอยู่ในช่วง ดังนั้นคำถามของฉันคือสนาม E และ B ของรังสีแม่เหล็กไฟฟ้าในเฟสหรือไม่? เราจะเข้าใจสิ่งนี้ได้อย่างไร


พลังงานอยู่ที่ไหนเมื่อทั้งสองฟิลด์กด 0 ในแผนภาพของคุณ ". ที่อื่น ๆ มันไม่เหมือนเขตข้อมูล E และ B ที่เป็น 0 ชั่วขณะ
MSalters

ในรูปสนามแม่เหล็กอยู่ในระนาบ XY ในขณะที่สนามไฟฟ้าอยู่ในระนาบ YZ (สมมติว่า Z ชี้ขึ้นไป) ดังนั้นจะไม่มีความแตกต่างของเฟส 90 องศาที่แสดงหรือไม่ โปรดแก้ไขถ้าฉันผิด
คนที่กระตือรือร้น

ดูที่physics.stackexchange.com/questions/461393/…มีภาพ 90 เฟสในคลื่น E และ B ในสถานการณ์นี้เราสามารถมีฟิลด์ "ส่งผลให้ E และ B" ไม่ได้อยู่ในเฟส แต่เกิดจากการเพิ่มคลื่นอีเอ็มสองตัว - ทั้งที่มี E และ B ในเฟส - แต่เดินทางไปในทิศทางตรงกันข้าม
Alejadro Xalabarder

คำตอบ:


8

ความสมบูรณ์ของสมการของแมกซ์เวลล์ทำให้ตำราเรียนในวิทยาลัยเต็มไปหมด

แต่เมื่อพิจารณาการแผ่รังสีจากเสาอากาศ (กระแสที่ไหลในตัวนำเชิงเส้น) มันจะเดือดลงไปตามความจริงที่ว่ามีองค์ประกอบที่แตกต่างกันมากมายในสนาม E (ไฟฟ้า) และ H (แม่เหล็ก) รอบ ๆ เสาอากาศ สำหรับฟิลด์ H มีองค์ประกอบหนึ่งที่ได้สัดส่วนกับ 1 / r 2และอีกองค์ประกอบหนึ่งที่เป็นสัดส่วน 1 / r สำหรับฟิลด์ E มีองค์ประกอบสามส่วนคือองค์ประกอบ1 / r 3ส่วนประกอบ 1 / r 2และส่วนประกอบ 1 / r

1 / r 3เทอมเป็นสนามไฟฟ้าไฟฟ้าสถิตไดโพลซึ่งหมายถึงพลังงานที่เก็บไว้ในสนาม capacitive เช่นเดียวกันคำว่า 1 / r 2หมายถึงพลังงานที่เก็บไว้ในสนามอุปนัย สิ่งนี้แสดงถึง "การเหนี่ยวนำตนเอง" ของตัวนำเสาอากาศซึ่งสนามแม่เหล็กที่ผลิตโดยกระแสเหนี่ยวนำให้เกิด "back EMF" บนตัวนำนั้นเอง เฉพาะคำว่า 1 / r เท่านั้นที่แสดงถึงพลังงานที่นำออกจากเสาอากาศ

ใกล้เสาอากาศซึ่งส่วนประกอบ1 / r 3และ 1 / r 2ครองความสัมพันธ์เฟสระหว่าง E และ H นั้นซับซ้อนและสาขาเหล่านี้จะเก็บพลังงานในลักษณะที่ Olin อธิบายและคืนพลังงานกลับไปที่เสาอากาศเอง .

อย่างไรก็ตามใน "สนามไกล" (เช่นมากกว่า 10 ความยาวคลื่นห่างจากเสาอากาศ) ส่วนประกอบ 1 / r ของสนามครองการสร้างคลื่นระนาบคลื่นแม่เหล็กไฟฟ้าและองค์ประกอบเหล่านี้อยู่ในขั้นตอนซึ่งกันและกัน


3
ฉันอยากเห็นคำอธิบายเพิ่มเติมเกี่ยวกับ 1 / r ^ 2 สำหรับฟิลด์ E และ H
Andy aka

คำถามหลักคือถ้าคลื่นแม่เหล็กไฟฟ้าระนาบ (แสดงในภาพ) มีองค์ประกอบของ EB ในเฟสหรือไม่ สิ่งที่เกิดขึ้นในสายอากาศเป็นเพียงหัวข้ออื่น
Alejadro Xalabarder

@AlejadroXalabarder: ไม่จริง คุณไม่สามารถส่งคลื่นแม่เหล็กไฟฟ้าได้หากไม่มีเสาอากาศบางประเภท "ระนาบคลื่น" เป็นเพียงมุมมองที่เรียบง่ายของสิ่งที่เกิดขึ้นใน "สนามไกล" ของเสาอากาศ นี่คือเหตุผลที่ฉันพยายามเชื่อมต่อระหว่างสิ่งที่แลงพูดกับสิ่งที่ OP อ่าน
Dave Tweed

@ เดฟ: เรามีระนาบคลื่นทุกที่ที่คุณรู้ว่าแสงตัวอย่างเช่น คุณกำลังพูดถึงสิ่งที่เกิดขึ้นในเสาวิทยุซึ่งเป็นกรณีเฉพาะของการสร้าง EM แต่ในกรณีนี้ใกล้กับเสาอากาศสมการแมกซ์เวลนั้นใช้ได้ดังนั้น E และ B อยู่ในเฟสเช่นกันแม้ว่าในกรณีนี้เราไม่มีคลื่นระนาบ จริงๆแล้วคำถามนี้เป็นเรื่องทั่วไปสำหรับคลื่น EM ทั้งหมดเท่านั้นที่จะเห็นปัญหาเฟสได้ง่ายขึ้นโดยใช้คลื่นระนาบที่พบบ่อยที่สุดและจริง
Alejadro Xalabarder

5

ความต้านทานของพื้นที่ว่างเป็นค่าคงที่ ค่าของมันคือสัดส่วนกับอัตราส่วนของ E และ H

เป็นปริมาณตัวต้านทานซึ่งหมายความว่า E และ H จะต้องเพิ่มขึ้นและลดลงในขนาดด้วยกัน

วิกิพีเดีย: - ป้อนคำอธิบายรูปภาพที่นี่


1
นี่คือกุญแจ ... ฉันจะเพิ่มรายละเอียดเล็กน้อย EXB จะยังคงให้ทิศทางในกรณีของความแตกต่างขั้นตอนเวลาที่เป็นจุดที่สำคัญคือว่า คุ้มค่าที่จะมีความซับซ้อน (ชิ้นส่วนจริงและจินตภาพ) - คือมี "การจัดเก็บข้อมูล" ปริมาณที่แท้จริงจะได้รับการต้านทาน
ตัวยึด

3

ความสับสนเกิดขึ้นจากความจริงที่ว่าพวกเขา (สนามแม่เหล็กไฟฟ้าและเวกเตอร์แม่เหล็ก) อยู่ห่างกัน 90 องศาในอวกาศไม่ใช่เวลา กล่าวคือ:

EB=0E×B


3

z^E=x^E0cos(ωtkz)

×E=tμH
HEHHออกจากเฟส แต่พวกเขาไม่ได้อยู่ในระนาบเดียวกันในพื้นที่อย่างใดอย่างหนึ่ง

โดยทั่วไปไดอะแกรมเช่นเดียวกับที่เชื่อมโยงกับคำถามอาจเป็นสิ่งที่ดีสำหรับการแสดงเขตข้อมูลในอวกาศและถ้าคุณดูอย่างรอบคอบคุณจะเห็นการยุติฟิลด์ การดูสมการอาจเป็นเพียงการเปิดเผยและถ้าคุณผ่านคณิตศาสตร์ Maxwell จะให้คำตอบ


2

ในการอ้างถึงWikipedia :

ชิ้นส่วนไฟฟ้าและสนามแม่เหล็กตั้งอยู่ในอัตราส่วนความแข็งแรงคงที่เพื่อตอบสนองสมการแมกซ์เวลสองตัวที่ระบุว่าจะผลิตหนึ่งจากอีกอันหนึ่งได้อย่างไร ฟิลด์ E และ B เหล่านี้ยังอยู่ในเฟสด้วยซึ่งจะถึง maxima และ minima ที่จุดเดียวกันในอวกาศ (ดูภาพประกอบ) ความเข้าใจผิดที่พบบ่อยคือสนาม E และ B ในการแผ่รังสีแม่เหล็กไฟฟ้าอยู่นอกเฟสเนื่องจากการเปลี่ยนแปลงในอันใดอันหนึ่งก่อให้เกิดอีกอันหนึ่งและสิ่งนี้จะสร้างความแตกต่างของเฟสระหว่างพวกมันในฐานะฟังก์ชันไซน์ - อยู่ใกล้กับเสาอากาศ)

คลื่นแม่เหล็กไฟฟ้า


เป็นหมายเหตุด้านทิศทางของเขตข้อมูลเหล่านี้กำหนดโพลาไรซ์ของสัญญาณ หากคุณเปลี่ยนแกนของฟิลด์ E และ B เสาอากาศบางประเภทจะไม่สามารถรับสัญญาณได้จนกว่าคุณจะหมุนเสาอากาศ 90 องศา (หรือบางชนิดของแว่นตากันแดดจะล้มเหลวในการส่งสัญญาณ)
ไบรอันดรัมมอนด์

1

แรงดันไม่ได้ขึ้นอยู่กับสนามแม่เหล็ก แต่ขึ้นอยู่กับอัตราการเปลี่ยนแปลง ดังนั้นแรงดันไฟฟ้าเหนี่ยวนำจึงสูงที่สุดเมื่อสนามแม่เหล็กเป็นศูนย์เมื่ออนุพันธ์ของมันสูงที่สุด

สำหรับพลังงานคงที่ในคลื่น EM เราต้องการส่วนประกอบแม่เหล็กและส่วนประกอบไฟฟ้าของแรงดันไฟฟ้าที่จะเป็น 90 องศาจากเฟส: ดังนั้นเราต้องการผลของสนามแม่เหล็กจะยิ่งใหญ่ที่สุดเมื่อสนามไฟฟ้าเป็น 0; สิ่งนี้จะเกิดขึ้นเมื่อฟิลด์อยู่ในเฟส


คุณไม่สับสนกับแรงดันไฟฟ้าเหนี่ยวนำในเสาอากาศกับสนาม E ของคลื่นแม่เหล็กไฟฟ้าหรือไม่? ในสุญญากาศ B = k̄ / c☓E (โดยมีทิศทางคลื่นk̄)
MSalters

1

ใช่พวกเขาอยู่ในเฟสหรือ -180 °เฟสตามที่ระบุโดย "Captainj2001" เมื่อใช้สมการแมกซ์เวลล์เพื่อสาธิต

EBH ) แต่ตอนนี้ฉันมั่นใจหลังจากทำตามเหตุผลของสมการแมกซ์เวล

EBE×B (พอยต์เวกเตอร์) ในทิศทางเดียวกัน และสิ่งนี้จะเกิดขึ้นเนื่องจากเวกเตอร์ Pointing นั้นสอดคล้องกับการแพร่กระจายของคลื่นเสมอ

ExByzE×B=z^ExByExByEB

โดยการใช้ไซต์ของเรา หมายความว่าคุณได้อ่านและทำความเข้าใจนโยบายคุกกี้และนโยบายความเป็นส่วนตัวของเราแล้ว
Licensed under cc by-sa 3.0 with attribution required.